PT68.3.18 - "The chairperson of Acme Corporation..."

Lime Green DotLime Green Dot Member
edited September 2021 in Logical Reasoning 1384 karma

Hello, 7Sagers!

Been awhile, hope everyone's easing into a nice LDW :)

I posted this on the question's discussion board but was hoping for a variety of views if possible, so I wanted to repost this here. Here's part of the post I was responding to:

...In answer choice E) however, look what happens if we negate it. Acme’s move to OV WILL be accompanied by a significant pay raise.

This is a necessary assumption because if we knew this specific point, then there is the POSSIBILITY of a gap in the argument.

The argument wouldn’t be 100% deducible that the workers won’t be able to afford the new location. That is also the reason why the LSAT writers specifically wrote SIGNIFICANT. Hope this helps

And my reply/question:

Hello, would you be able to clarify something?

I agree that the existence of a possible gap in the argument would obviously weaken it. But it seems to me that said gap could only be said to completely invalidate the argument if we knew the possible gap were an actual gap.

I know this may sound kind of fuzzy, but as an example, if for (E) it had said, “Acme’s move to OV will not be accompanied by a significant enough pay raise to allow Acme employees to afford housing in OV,” that to me would be a clear invalidator, a true gap that “destroys” the argument as we like to say around here.

But in negating (E) as it stands, we have, as you said, allowed for the mere possibility that the conclusion may not follow if we assume certain things about what (E) is getting at. For one thing, (E) is banking on the assumption I mentioned, and maybe others.

I don’t disagree that (E) is the best of the bunch, and the only one of the bunch that could remotely be considered the correct answer. But I guess the puzzling part is this sometimes hazy spectrum b/w “greatly weaken (w/o destroying)” and “absolutely destroy” and where a negated necessary assumption is allowed to fall within it. In my view [destroy → weaken], but the reverse is not necessarily true, and I just don’t see (E) meeting the “absolutely destroy” threshold…

This has been a recurring issue of mine & your help is appreciated!

Link to discussion: https://7sage.com/lsat_explanations/lsat-68-section-3-question-18/

🙏

Comments

  • canihazJDcanihazJD Alum Member Sage
    8318 karma

    "destroy the argument" ≠ conclusion cannot happen. In fact it still could. It just means the conclusion no longer follows from the premise.

  • Chris NguyenChris Nguyen Alum Member Administrator Sage 7Sage Tutor
    edited September 2021 4577 karma

    I'd argue here the LSAT is making us use what they call "common sense assumptions" with the word "significant"

    To the LSAT, what does it mean to be a "significant" pay raise? In LSAT terms, I read significant as important enough to be of note.

    With that in mind, I think answer choice (E) actually reads to me as what you said it should read if it were to invalidate the argument (Acme’s move to OV will not be accompanied by a significant enough pay raise to allow Acme employees to afford housing in OV).

    @"Lime Green Dot" said: But I guess the puzzling part is this sometimes hazy spectrum b/w “greatly weaken (w/o destroying)” and “absolutely destroy” and where a negated necessary assumption is allowed to fall within it.

    To answer this question, I think a correct NA question will mean that the conclusion will not follow 100%. Weakening doesn't cut it. With the common sense assumption the LSAT is making us assume here regarding the word "significant", the negation of (E) doesn't allow the conclusion to follow.

    Hope this helps!

  • Lime Green DotLime Green Dot Member
    edited September 2021 1384 karma

    Thank you both! @canihazJD and @Christopherr. I appreciate having 2 tutors' feedback greatly!

    A couple of more comments:

    Personally, @Christopherr, I feel compelled to argue as others have on this question's comment section that "significant" is inherently subjective: while it does imply "a large amount," how are we to know whether that large amount is the 'enough' we need to equate it with the employees being able to afford housing in OV? I find the distinction particularly important to make here, as having a big pay raise and being able to suddenly afford housing where one could not before may or may not line up with each other. They could be on completely different scales, I think (ex: if I made $10/hour in M, got a 'significant raise' to $20/hour in OV, I still may not be able to afford housing around there if the starting range of housing is, say, around half a million dollars), which is why I believed the fully fleshed out version of (E) would have been a necessary addition.

    @Christopherr said:

    With the common sense assumption the LSAT is making us assume here regarding the word "significant", the negation of (E) doesn't allow the conclusion to follow.

    On the other hand, if we are to take the "common sense assumption" approach as you suggest, and it encompasses the fuller version of (E) I wrote out, I'd feel much more at ease with my understanding of both this Q and NA Qs in general that are similar to this one.

    @canihazJD said:

    In fact it still could. It just means the conclusion no longer follows from the premise.

    I still see a distinction between a possibility gap and an absolute gap. I feel (E) is an example of the former. Just wanted to ask you whether "may no longer follow" from the premise (possibility gap) implies "no longer follows" (absolute gap) from the premise, or whether you see it in this way. I do see that possibility element in the conclusion portion (even with the addition of [E], the conclusion still may or may not be true), but I also see it existing from the premise → conclusion structure (even with the addition of [E], the conclusion may or may not follow from the premise). This must be a fairly rudimentary question, but is there a way you distinguish between these two?

    Many thanks! :)

  • canihazJDcanihazJD Alum Member Sage
    edited September 2021 8318 karma

    @"Lime Green Dot"

    I still see a distinction between a possibility gap and an absolute gap. I feel (E) is an example of the former. Just wanted to ask you whether "may no longer follow" from the premise (possibility gap) implies "no longer follows" (absolute gap) from the premise, or whether you see it in this way. I do see that possibility element in the conclusion portion (even with the addition of [E], the conclusion still may or may not be true), but I also see it existing from the premise → conclusion structure (even with the addition of [E], the conclusion may or may not follow from the premise). This must be a fairly rudimentary question, but is there a way you distinguish between these two?

    Well there is a distinction between a gap that is possible and a gap that actually occurs, it just doesn't matter for the purposes of this question. To me it sounds kind of like you are conflating, "may no longer follow" with "may not happen."

    we're moving the shop
    most people cant afford to live within a short commute
    so most people will have a long commute

    Makes reasonable sense.

    we're moving the shop
    most people cant afford to live within a short commute
    but we're going to give them all a huge raise
    so most people will have a long commute

    This does't make sense as an argument anymore, despite the fact that it could very well be true.

    Another way is to focus on the blocking/defending role of the answer choice. By saying they didn't give a significant raise, it is precluding one way (of several available) of providing for a closer commute. In this case, if they didn't give a significant raise, then they didn't give a raise that would cover the cost of relocation.

    Another correct answer might be something like, there's not a new freeway or form of mass transit that drastically reduces commute time. You wouldn't have to specify that it would reduce commute time to under 30 minutes... just the fact that it doesn't drastically reduce, if you add its negation to the argument, causes the conclusion to no longer follow.

    I'll also add in here:

    Personally, @Christopherr, I feel compelled to argue as others have on this question's comment section that "significant" is inherently subjective: while it does imply "a large amount," how are we to know whether that large amount is the 'enough' we need to equate it with the employees being able to afford housing in OV?

    It is absolutely subjective. But again it doesn't matter. Maybe a large amount wouldn't have been enough... but if they gave a "significant" amount, it no longer follows that most people would still not be able to afford a move. Sure it could not be enough, but to conclude that it isn't is unsupported. Conversely, if they don't give a significant amount, then they likely don't give enough to finance a move, so the argument survives.

    Aside, IMO additional nomenclature and stress over subjective meanings is unproductive if not counterproductive. It's usually the product of misunderstanding the mechanics of the question. The answer almost always is that it doesn't matter. It usually behooves us to find less to worry about... the test is hard without us artificially inflating the difficulty.

  • Lime Green DotLime Green Dot Member
    1384 karma

    Well there is a distinction between a gap that is possible and a gap that actually occurs, it just doesn't matter for the purposes of this question. To me it sounds kind of like you are conflating, "may no longer follow" with "may not happen."

    Yes, that is what happened.

    we're moving the shop
    most people cant afford to live within a short commute
    so most people will have a long commute

    Makes reasonable sense.

    we're moving the shop
    most people cant afford to live within a short commute
    but we're going to give them all a huge raise
    so most people will have a long commute

    This does't make sense as an argument anymore, despite the fact that it could very well be true.

    Maybe a large amount wouldn't have been enough... but if they gave a "significant" amount, it no longer follows that most people would still not be able to afford a move. Sure it could not be enough, but to conclude that it isn't is unsupported.

    Okay, this clarified a whole LOT for me. I think I get it now! ヽ(•‿•)ノ

    So, just to put it in me-speak, while the conclusion may or may not happen following the insertion of "but we're going to give them all a huge raise," the conclusion no longer follows (not just 'may not' follow, if I'm understanding correctly, but verily does not follow) after that insertion. It simply doesn't make sense to assert this line of reasoning anymore in spite of the possibility that the conclusion may still independently happen.

    Aside, IMO additional nomenclature and stress over subjective meanings is unproductive if not counterproductive. It's usually the product of misunderstanding the mechanics of the question.

    Sigh.. yes, that is once again what happened here. I guess I led myself down a dead-end on this one, but there have been times when the LSAT has tempted me into an unwarranted assumption that hinged on a subjective interpretation like this. Thought this was one of those times, but I guess I just gotta be more wary about when it matters and when it doesn't. Anywho, thanks again, @canihazJD!!

  • WinningHereWinningHere Member
    417 karma

    But the argument is concluding that the employees won't afford the move, so if they do "not" get a raise, that supports the conclusion.

  • hotranchsaucehotranchsauce Member
    edited September 2021 288 karma

    In my opinion:

    1) Define Game Type:

    Necessary Assumption: Choose an answer that, when presented into the argument in its negated form, will destroy the argument.

    2) Reduce argument

    The chairperson of Acme Corporation has decided to move the company from its current location in Milltown to Ocean View. Most Acme employees cannot afford housing within a 30-minute commute of Ocean View. So once the company has moved, most Acme employees will have a commute of more than 30 minutes.

    Most employees cannot afford housing. So once moved, most commute more.

    3) Translate into caveman

    Most can't buy house. So, if move commute more.

    (P) most can't buy house
    (CC) If move, more commute

    Here's answer choice E, translated into caveman and negated:

    (P) most can't buy house
    E: ALL EMPLOYEES HUGE MONEY
    (CC) If move, more commute

    E can't be in the argument when negated, and must be in the argument in its presented form.

    If you go about it in hindsight, you can see this problem is just testing your ability to see that answer choice e, when negated, hugely conflicts with the stated premise. You can even ignore the CC altogether.

    (P) most can't buy house
    (P) all employees huge money
    They just can't live well together. Of course these are boiled down into "caveman", but even if you're 75% accurate in your caveman translations I think you'll still maintain the meaning enough to arrive at the correct answer choice.

    Here are some other translations that would work:

    Not enough $, even more$
    house too $, more $
    NOT $, YES $ (probably the most fundamental this can be broken down if you filter everything down into caveman)

    Seeing such a big disconnect between a stated premise and a negated answer choice... well I think you just found the correct answer.

    Just doing a quick negation scan in caveman of the other answer choices
    (--X-> means not necessarily) :

    A: cannot afford, awesome works with "can't buy house"
    B: chairperson --X-> reasons, awesome works with "can't buy house"
    C: - in favor or in favor, awesome works with "can't buy house"
    D: -currently (does not mean future), awesome works with "can't buy house" (this negation is tricky for me i'm still not quite sure how to negate it, but I don't think you need to)
    E: HUGE MONEY

Sign In or Register to comment.